Line integral, problems with substitution (probably)

Click For Summary
SUMMARY

The discussion revolves around solving the line integral \(\int_C 4x^3 dS\) where the curve \(C\) is defined by the parametric equations \(x=t\) and \(y=t^3-1\) for \(0 \leq t \leq 1\). The user initially miscalculated the integral, arriving at an incorrect answer of 2 instead of the correct value of approximately 2.69. The error was identified as a mistake in calculating the new limits after substitution, which should have been 10 and 1 instead of 0 and 9.

PREREQUISITES
  • Understanding of line integrals in calculus
  • Familiarity with parametric equations
  • Knowledge of substitution methods in integration
  • Ability to compute limits of integration
NEXT STEPS
  • Study the properties of line integrals in vector calculus
  • Learn about parametric curves and their applications
  • Practice substitution techniques in definite integrals
  • Explore numerical methods for approximating integrals
USEFUL FOR

Students learning calculus, particularly those focused on integration techniques and line integrals, as well as educators seeking to clarify common mistakes in solving integrals.

Saraphim
Messages
46
Reaction score
0

Homework Statement



I am trying to solve a line integral (bear with me, I am new to calculus!) and my basic skills of integration seem to fail me. I am sure the mistake is quite obvious, as I keep getting the wrong answer, 2, when it should be ~2.69

Homework Equations


\int_C 4x^3dS

C is the curve given by x=t, y=t^3-1 and 0 \leq t \leq 1

The Attempt at a Solution


\int_C 4x^3 dS = \int_0^1 4t^3 \sqrt{1^2+(3t^2)^2} dt = \int_0^1 4t^3 \sqrt{9t^4+1} dt
I attempt substitution in order to solve the integral:
u=9t^4+1 \Rightarrow \frac{1}{36}du=t^3 dt.
The limits are now 9 \cdot 0^4=0 and 9 \cdot 1^4=9, so by substitution we have:
\frac{4}{36} \int_0^9 \sqrt{u} \, du = \frac{1}{9} \left[\frac{2}{3} u^{\frac{3}{2}}\right]_0^9 = \frac{2}{27} \left(9^{\frac{3}{2}} - 0\right)

Which equals... Two! Well, there's a mistake in there somewhere, so, no it doesn't. Hopefully a non-mathematician will have mercy on me so that the punishment isn't too hard. o:)

-- Sarah
 
Last edited:
Physics news on Phys.org
Nevermind, I found the problem. I did not correctly calculate the new limits, they should be 10 and 1 respectively. :biggrin:

-- Sarah
 
Question: A clock's minute hand has length 4 and its hour hand has length 3. What is the distance between the tips at the moment when it is increasing most rapidly?(Putnam Exam Question) Answer: Making assumption that both the hands moves at constant angular velocities, the answer is ## \sqrt{7} .## But don't you think this assumption is somewhat doubtful and wrong?

Similar threads

  • · Replies 105 ·
4
Replies
105
Views
6K
  • · Replies 2 ·
Replies
2
Views
1K
Replies
12
Views
2K
  • · Replies 7 ·
Replies
7
Views
2K
  • · Replies 10 ·
Replies
10
Views
2K
Replies
3
Views
2K
  • · Replies 8 ·
Replies
8
Views
2K
Replies
4
Views
2K
  • · Replies 8 ·
Replies
8
Views
2K
  • · Replies 1 ·
Replies
1
Views
2K